0 Daumen
175 Aufrufe

Aufgabe:

Beweise, dass

\( \frac{n^{n}}{3^{n}} \leq n! \)

für alle n \(\geq\) 6 gilt.


Problem/Ansatz:

Ich habe Induktion probiert, komme aber zu keinem Ergebnis.

Avatar von

n^n/3^n = (n/3)^n

Man könnte die n-te Wurzel ziehen

Vlt. hilft das weiter.

Woher stammt die Aufgabe. Kaum ein Dozent würde die Bedingung n >= 6 dazuschreiben, wenn sie nicht nötig ist. Wurde die Aufgabe fehlerfrei abgeschrieben?

1 Antwort

+2 Daumen
 
Beste Antwort

Die Ungleichung gilt für alle \(n\in \mathbb N\).
Wir wissen, \(e<3\) und \(e^n = \sum_{k=0}^\infty \frac{n^k}{k!}\).

\( \frac{n^{n}}{3^{n}} \leq n! \Leftrightarrow \frac{n^{n}}{n!} \leq 3^{n}\)

Nun gilt

\(\frac{n^{n}}{n!} \leq \sum_{k=0}^\infty \frac{n^k}{k!} = e^n \leq 3^n\)

Fertig.

Nachtrag - Induktion:

\(n=1\) klar.

\(n\rightarrow n+1\):

\(\frac{(n+1)^{n+1}}{3^{n+1}} = \frac{n^{n}}{3^{n}}\cdot (n+1) \frac 13 \left(1+\frac 1n\right)^n \)

\(\stackrel{\text{Induktionsvorausssetzung}}{\leq} (n+1)!\cdot \frac 13 \left(1+\frac 1n\right)^n \)

Es genügt also zu zeigen, dass \(\left(1+\frac 1n\right)^n < 3\).

\(\left(1+\frac 1n\right)^n =  1+1 +\sum_{k=2}^n \underbrace{\binom nk \frac 1{n^k}}_{\stackrel{\leq \frac 1{k!}}{\text{Hausaufgabe}}  } < 2+ \sum_{k=2}^{\infty} \frac 1{2^k}  = 3\)

Nochmal fertig.


Avatar von 10 k

Wie kommt man auf diesen genialen Ansatz?

Gäbe es Alternativen?

Ein anderes Problem?

Stell deine Frage

Willkommen bei der Mathelounge! Stell deine Frage einfach und kostenlos

x
Made by a lovely community